Julien Santini wrote:
> Allez, step by step ... si cette fois-ci vous comprenez pas -ou ne me
> montrez pas mon erreur éventuelle, bien sûr-, je raccroche.
>
> ==
> Soit O un ouvert de R, réunion d'ouverts (O_i) (i in I, I non vide).
> ==
>
> Pour tout i in I, il existe une famille dénombrable d'intervalles ouverts
> I_(i,k) de R centrés en des rationnels et de rayons rationnels tels que
> O_i = Réunion(I_(i,k); k in Q).
> On a alors: O = Réunion(O_i, i in I) = Réunion(I_(i,k); k in Q, i in I).
> Mais il n'existe qu'un nombre dénombrable de I_(i,k) car Q x Q est
> dénombrable (et un intervalle I_(i,k) s'identifie au couple (a,b) donnant
> son centre et son rayon), donc la réunion précédente est quoiqu'il arrive
> une réunion dénombrable (plusieurs mêmes intervalles intervenant peut-être
> plusieurs fois).
> Maintenant on renumérote les I_(i,k) en indexant sur t in Q: O =
> Réunion(I_t, t in Q). Pour tout t dans Q, il existe un O_t contenant I_t
> (par construction des I_(i,k)). Alors O = Réunion(O_t, t in Q).Je n'avais pas vu dans tes messages precedents la mention concernant les
rayons qui devaient etre rationnels

Effectivement, comme ca, ca marche.
Je poste le corrige que j'ai fait de cet exercice, avec l'aide de collegue
(et la votre, bien entendu

; c'est en LaTeX perso, donc pas compilable
directement (sorry), j'espere que c'est lisible. Il y a un eclairage
interessant a la fin. Si vous connaissez le nom de ce theoreme, ou si vous
avez le Bourbaki en question sous la main, je suis interesse.
\begin{Ex}
\begin{fmpage}{0.95\textwidth}
Soit $(\Omega_i)_{i\in I}$ une famille quelconque d'ouverts de
$\R$. Montrer qu'il existe une partie $J$ de $I$ \emph{au plus
dénombrable} telle que $\bigcup_{i\in J}\Omega_i = \bigcup_{i\in
I}\Omega_i$.
\end{fmpage}
\begin{solution} :
L'ensemble $\Q$, par son caractère dénombrable et sa densité dans
$\R$, va nous être d'un grand secours. Notons
$\Omega=\bigcup_{i\in I}\Omega_i$. L'idée de la démonstration
consiste à recouvrir $\Omega$ par une famille \emph{dénombrable}
d'intervalles ouverts extraits des $\Omega_i$. On verra en fin
d'exercice une explication profonde à ce phénomène, et quelques
pistes plus ou moins fructueuses à essayer.
Notons $\Theta=\left\{(x,r)\in\Q\times\Q^{\ast+}\tq (\exists\,i\in
I)\,]x-r,x+r[\subset \Omega_i\right\}$, et $\phi$ une fonction
de choix\footnote{Du coup, l'axiome du choix semble inévitable,
mais c'est souvent le cas lorsqu'on manipule des ensembles
infinis !} permettant d'associer à chaque couple
$(x,r)\in\Theta$ un indice $i=\phi(x,r)$ tel que
$]x-r,x+r[\subset\Omega_i$.
La famille $(\Omega_{\phi(x,r)})_{(x,r)\in\Theta}$ est
dénombrable, puisque indexée par un sous-ensemble de
$\Q\times\Q^{\ast+}$ qui l'est. Notant
$\Omega'=\bigcup_{(x,r)\in\Theta} \Omega_{\phi(x,r)}$, on a
clairement $\Omega'\subset\Omega$. Reste à montrer l'inclusion
réciproque.
Il est tout aussi clair que $\Omega\cap\Q\subset\Omega'$. En
effet, si $x\in\Omega\cap\Q$, $x$ appartient à l'un des
$\Omega_i$, qui en est un voisinage. Il existe donc $\alpha>0$ tel
que $]x-\alpha,x+\alpha[$ soit inclus dans l'un des $\Omega_i$, et
quitte, grâce à la densité de $\Q$ dans $\R$, à réduire un peu
$\alpha$, on peut supposer qu'il est rationnel. Ainsi,
$(x,\alpha)\in\Theta$ et $x\in\Omega_{\phi(x,\alpha)} \subset
\Omega'$.
Considérons maintenant le cas d'un irrationnel
$x\in\Omega\setminus\Q$. $x$ appartient à l'un des $\Omega_i$, qui
en est un voisinage, il existe donc un $\beta>0$ tel que
l'intervalle $]x-\beta,x+\beta[$ soit contenu dans l'un des
$\Omega_i$, et donc dans $\Omega$. Mais, toujours par densité de
$\Q$, l'intervalle $]x-\epsilon,x[$ (resp. $]x,x+\epsilon[$)
contient un rationnel $q'$ (resp. $q''$).
Notant $q=\frac{q'+q''}{2}$ et $r=\frac{q''-q'}{2}$ (qui sont tous
deux rationnels !) , d'une part $x\in]q-r,q+r[$, d'autre part
$]q-r,q+r[\subset \Omega_{\phi(q,r)}\subset\Omega'$. On conclut
donc encore que $x\in\Omega'$.
Finalement, on a démontré que $\Omega\cap\Q\subset\Omega'$ et
$\Omega\setminus\Q\subset\Omega'$, donc
$$\Omega=(\Omega\cap\Q)\cup(\Omega\setminus\Q)\subset\Omega'$$
Voici une généralisation\footnote{qui m'a été signalée par Hubert
\textsc{Quatreville}, et dont la démonstration, qui est une
simple généralisation de celle présentée ci-dessus, peut être
trouvée dans ``Bourbaki Topologie générale Chapitre IX,
Appendice 1 : Espace de Lindelöf''.} qui apporte un éclairage
sur la magie apparente de cette démonstration :
\begin{theoreme}[Nom du théorème]
Soit $E$ un espace topologique admettant une \emph{base
dénombrable d'ouverts}. Alors de toute famille
$(\Omega_i)_{i\in I}$ d'ouverts, on peut extraire une
sous-famille \emph{dénombrable} $(\Omega_j)_{j\in J}$ telle que
$\bigcup_{j\in J}\Omega_j=\bigcup_{i\in I}\Omega_i$.
\end{theoreme}
Une autre approche\footnote{qui m'a été signalée par Romain
\textsc{Krust}, toujours sur l'excellente liste de diffusion de
l'UPS !} consiste à construire une \emph{suite croissante de
compacts} $(K_n)_{n\in\N}$ dont la réunion est
$\Omega$\footnote{On peut considérer par exemple
$K_n=\left\{x\in\Omega\tq d(x,\complement_\R
\Omega)\ge\frac{1}{n}\right\}\cap[-n,n]$.}. Chacun de ces
compacts est recouvert, d'après la propriété de Borel-Lebesgue,
par un nombre fini des $\Omega_i$. La réunion de ces recouvrements
finis constitue un recouvrement dénombrable de $\Omega$.
\end{solution}
\end{Ex}
Merci pour votre patience et votre perseverance a tous deux. J'espere que ca
vous a amuse (j'ai pas l'impression, pour Pierre...) ou au moins interesse.
\bye
--
Nicolas FRANCOIS
http://nicolas.francois.free.frWe are the Micro$oft.
Resistance is futile.
You will be assimilated.